nima1376

New Member
ارسال ها
218
لایک ها
93
امتیاز
0

aras2213

New Member
ارسال ها
216
لایک ها
228
امتیاز
0
پاسخ : ماراتن نظریه ی اعداد (سطح ممتاز)

ثابت کنید اگر
آنگاه برای بی نهایت n
.
 

math

New Member
ارسال ها
1,129
لایک ها
1,096
امتیاز
0
پاسخ : ماراتن نظریه ی اعداد (سطح ممتاز)

ثابت کنید اگر
آنگاه برای بی نهایت n
.
سوال مرحله 3 ایران بوده .

راه حل اول : با استقرا ثابت کنید که تمام توان های 2 جوابند .

راه حل دوم :
رابطه ی صریح یه دنباله بازگشتی هستش که با 3 جمله ی اول معلوم میشه و .....
 

nima1376

New Member
ارسال ها
218
لایک ها
93
امتیاز
0
پاسخ : ماراتن نظریه ی اعداد (سطح ممتاز)

سوال بعدی
تمام چند جمله ای های pرا بیابید به طوری که
 

math

New Member
ارسال ها
1,129
لایک ها
1,096
امتیاز
0
پاسخ : ماراتن نظریه ی اعداد (سطح ممتاز)

سوال بعدی
تمام چند جمله ای های pرا بیابید به طوری که

این هم سوال مرحله 3 ایران سال 2004 بوده .

اگر
انگاه چون
پس تناقض است پس برای هر عدد اول داریم :
حال قرار دهید

این حرف ها در باره ی Q نیز درست است . پس تنها جواب مساله
است
 
آخرین ویرایش توسط مدیر

nima1376

New Member
ارسال ها
218
لایک ها
93
امتیاز
0
پاسخ : ماراتن نظریه ی اعداد (سطح ممتاز)

سوال بعدی
نشان دهید در دنباله
هیچ عددی بی نهایت بار نمی آید
فقط من نباید سوال قرار بدم آقای math شماهم یه سوالی قرار بدید
 

n_maths

New Member
ارسال ها
322
لایک ها
275
امتیاز
0
پاسخ : ماراتن نظریه ی اعداد (سطح ممتاز)

وقتی سوال بالاییه حل شد،لطفا این سوال رو هم حل کنید
تمام اعداد صحیح مثبت فرد بزرگتر از n>1 را پیدا کنید به طوری که برای هر مقسوم علیه نسبت به هم اول a,b از n ،عدد a+b-1 عدد n را عاد کند (روسیه2001)
 

nima1376

New Member
ارسال ها
218
لایک ها
93
امتیاز
0
پاسخ : ماراتن نظریه ی اعداد (سطح ممتاز)

وقتی سوال بالاییه حل شد،لطفا این سوال رو هم حل کنید
تمام اعداد صحیح مثبت فرد بزرگتر از n>1 را پیدا کنید به طوری که برای هر مقسوم علیه نسبت به هم اول a,b از n ،عدد a+b-1 عدد n را عاد کند (روسیه2001)
کوچکترین عامل آن را بگیرید
a,b را همان کوچکترین عامل و ضرب بقیه عوامل قرار میدهیم
نتیجه مشود n یک عامل دارد
 

amirhossein1376

Well-Known Member
ارسال ها
284
لایک ها
272
امتیاز
63
پاسخ : ماراتن نظریه ی اعداد (سطح ممتاز)

سوال بعدی:

اگر p,q اعداد طبیعی باشند که:

ثابت کنید p به وسیله ی 1979 عاد میشود.

البته الان سال این سوال مشخص شد ولی لطفا روش فکر کنید بعدا جوابشو پیدا کنید
 

math

New Member
ارسال ها
1,129
لایک ها
1,096
امتیاز
0
پاسخ : ماراتن نظریه ی اعداد (سطح ممتاز)

سوال بعدی:

اگر p,q اعداد طبیعی باشند که:

ثابت کنید p به وسیله ی 1979 عاد میشود.

البته الان سال این سوال مشخص شد ولی لطفا روش فکر کنید بعدا جوابشو پیدا کنید

ضرب کنید و با استفاده از قضیه ویلسون و اینکه 1979 عددی اول است مساله رو حل کنید .

خوب به دستور استاد نیما این هم سوال بعد :4:


ثابت کنید بی نهایت عدد طبیعی n وجود دارد که :


که در آن
تعداد مقسوم علیه های n است .
 

n_maths

New Member
ارسال ها
322
لایک ها
275
امتیاز
0
پاسخ : ماراتن نظریه ی اعداد (سطح ممتاز)

کوچکترین عامل آن را بگیرید
a,b را همان کوچکترین عامل و ضرب بقیه عوامل قرار میدهیم
نتیجه مشود n یک عامل دارد
یکم بیشتر توضیح میدین؟
 

math

New Member
ارسال ها
1,129
لایک ها
1,096
امتیاز
0
پاسخ : ماراتن نظریه ی اعداد (سطح ممتاز)

وقتی سوال بالاییه حل شد،لطفا این سوال رو هم حل کنید
تمام اعداد صحیح مثبت فرد بزرگتر از n>1 را پیدا کنید به طوری که برای هر مقسوم علیه نسبت به هم اول a,b از n ،عدد a+b-1 عدد n را عاد کند (روسیه2001)
میتونید این جا رو ببینید :3:

AoPS Forum - Divisors a, b, a+b-1 of n • Art of Problem Solving
 

aras2213

New Member
ارسال ها
216
لایک ها
228
امتیاز
0
پاسخ : ماراتن نظریه ی اعداد (سطح ممتاز)

ضرب کنید و با استفاده از قضیه ویلسون و اینکه 1979 عددی اول است مساله رو حل کنید .

خوب به دستور استاد نیما این هم سوال بعد :4:


ثابت کنید بی نهایت عدد طبیعی n وجود دارد که :


که در آن
تعداد مقسوم علیه های n است .
اون داخل پرانتز یه چند جمله ایه که بر بی نهایت عدد اول بخش پذیره بعد با باقیمانده چینی یه x وجود داره که این چند جمله ای به ازای x به تعداد زیادی عدد اول بخشپذیره

---- دو نوشته به هم متصل شده است ----

سوال بعد:
فرض کنید p عددی اول باشد.تمام اعداد صحیح k را بیابید که
عددی طبیعی باشد.
سوال بعدی
نشان دهید در دنباله
هیچ عددی بی نهایت بار نمی آید
فقط من نباید سوال قرار بدم آقای math شماهم یه سوالی قرار بدید
در پست های قبلی آقای mahanmath پاسخ داده اند
 
آخرین ویرایش توسط مدیر

nima1376

New Member
ارسال ها
218
لایک ها
93
امتیاز
0
پاسخ : ماراتن نظریه ی اعداد (سطح ممتاز)

اون داخل پرانتز یه چند جمله ایه که بر بی نهایت عدد اول بخش پذیره بعد با باقیمانده چینی یه x وجود داره که این چند جمله ای به ازای x به تعداد زیادی عدد اول بخشپذیره

---- دو نوشته به هم متصل شده است ----

سوال بعد:
فرض کنید p عددی اول باشد.تمام اعداد صحیح k را بیابید که
عددی طبیعی باشد.

در پست های قبلی آقای mahanmath پاسخ داده اند
حالت 1

حالت 2

جالبه اگه سوال به شکل
بود سوالی open بود




تو کدوم صفحه راه حل سوال من داده شده؟



سوال بعد معادله زیر را حل کنید
 
آخرین ویرایش توسط مدیر

aras2213

New Member
ارسال ها
216
لایک ها
228
امتیاز
0
پاسخ : ماراتن نظریه ی اعداد (سطح ممتاز)

حالت 1

حالت 2

جالبه اگه سوال به شکل
بود سوالی open بود





تو کدوم صفحه راه حل سوال من داده شده؟



سوال بعد معادله زیر را حل کنید
صفحه ی 8 پست 76:91:
 
آخرین ویرایش توسط مدیر

amirhossein1376

Well-Known Member
ارسال ها
284
لایک ها
272
امتیاز
63
پاسخ : ماراتن نظریه ی اعداد (سطح ممتاز)

منم نمیفهمم آخه یعنی چی لطفا اگه خودتون چیزی فهمیدین اینجا بذارید:24:
 

aras2213

New Member
ارسال ها
216
لایک ها
228
امتیاز
0
پاسخ : ماراتن نظریه ی اعداد (سطح ممتاز)

لطفا آدرس ندید لااقل جوابو اینجا بذارید
این آدرسی که من دادم مال سوال قبلی آقای nima1376 هستش که خودم هم نمی فهمم اون جا چی نوشته .:179:
 

amirhossein1376

Well-Known Member
ارسال ها
284
لایک ها
272
امتیاز
63
پاسخ : ماراتن نظریه ی اعداد (سطح ممتاز)

پس ببخشید من فکر کردم مال این سواله
 
بالا